Foro de preguntas y respuestas de matemáticas, de cualquier nivel. Cuánto más interesantes, divertidas o intrépidas, mejor.
Aviso: Te invitamos a conocer la página de Facebook de la UCIM

Ganas puntos al hacer preguntas, contestarlas y, sobre todo, si tu respuesta es seleccionada como la mejor.
Registrate como usuario para participar en el foro. También puedes utilizar tu identidad de FB Utiliza el botón azul para ingresar (si usas tu identidad de FB y estás logeado en FB, automáticamente te reconoce).

El irracional tiene una página en FB. El Irracional






+5 votos
Si existe una biyección entre la potencia de $A$ y la potencia de $B$ ¿Se puede asegurar que $A$ y $B$ tienen la misma cardinalidad?

En otras palabras ¿hay logaritmo en los cardinales?

No se me ocurre cómo probarlo o refutarlo. Mi conjetura es que seguramente es un resultado conocido o es de esas cosas que no se pueden probar ni refutar. Espero alguien sepa.
por (8,1m puntos) en Preguntas
El siguiente enlace podría serte de ayuda: http://goo.gl/ViRhRf
Lo es sin duda. :)
Me habría gustado que fuera cierto y con una demostración accesible, pero pues así son las cosas.

Ahora no me queda claro si es equivalente a la hipótesis del continuo generalizada, o dónde está parada esta afirmación con respecto a la hipótesis del continuo y la misma generalizada.
No hay ninguna relación "fácil" entre tu enunciado y la hipótesis generalizada del continuo. Por ejemplo, es fácil (si se sabe "forcing") producir modelos donde, por ejemplo, $2^{\aleph_0}=2^{\aleph_1}=\cdots=2^{\aleph_n}=\cdots=\aleph_{\omega+1}$ pero $2^{\aleph_\alpha}=\aleph_{\alpha+1}$ para $\alpha\geq\omega$. Si buscas información (en Google, o pregúntale a alguien que sepa) sobre el Teorema de Easton entonces quizá te quede más claro esto...
Licencia Creative Commons
Este obra está bajo una Licencia Creative Commons Atribución-NoComercial-CompartirIgual 2.5 México.

powered by UCIM  -  Aviso de privacidad

...